Si Fermat avait la solution..
Répondre à la discussion
Page 1 sur 3 12 DernièreDernière
Affichage des résultats 1 à 30 sur 83

Si Fermat avait la solution..



  1. #1
    leg

    Si Fermat avait la solution..


    ------

    bonjour
    P de fermat , prétendait avoir la solution de son théorème xN+YN=ZN,n'à pas de solution pour N > 2.

    Pourtant malgré les différentes remarques de la communauté mathématique; tous, pense qu'il a pu se tromper mais personne ne semble savoir qu'elle aurait été son idée, qui lui a fait dire qu'il avait une démonstration ...etc, vu l'époque elle ne pouvait être qu'élémentaire.

    Je vous soumets donc ce qui je pense à du être son idée et qui pourrait bien lui donner raison.
    Son cheval de bataille était les triplets Pythagoriciens.
    Je joins le fichier, si ça marche.

    -----

  2. #2
    leg

    Re : Si Fermat avait la solution..

    je ne comprend pas pourquoi je ne peux joindre le fichier text.
    si quequ'un à la réponse, merci

  3. #3
    inviteae1ed006

    Re : Si Fermat avait la solution..

    Bonjour Leg,
    pour joindre un fichier, tu dois le mettre quelque part sur internet (sur un site par exemple) et ensuite mettre le lien vers ce site :
    exemple :
    http://www.monsite.fr

  4. #4
    invite986312212
    Invité

    Re : Si Fermat avait la solution..

    il me semble qu'Edwards, dans son bouquin sur le théorème de Fermat, émet des hypothèses sur la démonstration "merveilleuse mais trop longue..etc". Elle pourrait être fondée sur sa méthode de "descente" comme sa démonstration pour le cas N=4. Ce n'est qu'une hypothèse bien sûr.

  5. A voir en vidéo sur Futura
  6. #5
    leg

    Re : Si Fermat avait la solution..

    Citation Envoyé par ambrosio Voir le message
    il me semble qu'Edwards, dans son bouquin sur le théorème de Fermat, émet des hypothèses sur la démonstration "merveilleuse mais trop longue..etc". Elle pourrait être fondée sur sa méthode de "descente" comme sa démonstration pour le cas N=4. Ce n'est qu'une hypothèse bien sûr.
    bonjour ambrosio
    justement sa descente infinie est une fauuse piste pour généraliser elle ne peut résoudre que le cas par cas et pas tous.

    prgasp va mettre le fichier sur le forum, puis on pourra discuter et avoir des avis.

  7. #6
    leg

    Re : Si Fermat avait la solution..

    http://yankee.sierra77.free.fr/perso...__Fermat_1.doc
    le fichier , pourrais tu le mettre sur le forum en piece jointe,gasp77. merci leg

  8. #7
    leg

    Re : Si Fermat avait la solution..

    ton site prgasp77 n'est pas valid, impossible de voir le fichier.

  9. #8
    leg

    Re : Si Fermat avait la solution..

    en espérant que çà marche:
    Images attachées Images attachées

  10. #9
    GuYem

    Re : Si Fermat avait la solution..

    Réponse qui ne sert à rien dans la discussion : ça marche !
    Bravo jolie Ln, tu as trouvé : l'armée de l'air c'est là où on peut te tenir par la main.

  11. #10
    prgasp77

    Re : Si Fermat avait la solution..

    Citation Envoyé par leg Voir le message
    ton site prgasp77 n'est pas valid, impossible de voir le fichier.
    Heu ... tu as entre-coupé ton lien par des points de suspentions ... Alors oui le lien n'était plus valide

    Enfin, tu as fini par trouver comment joindre une pièce
    Je lis ça et je dis ce que j'en pense.
    --Yankel Scialom

  12. #11
    Médiat

    Re : Si Fermat avait la solution..

    Loin de moi l'idée d'affirmer sans preuve que cette démonstration est invalide, mais je dois avouer que je suis incapable de la lire ; par exemple, la première phrase est :

    Où choisir un couple de paramètres p et q, formant un triplet de racines carrées :
    x’ , y’, z’ vérifiant l’égalité X + Y = Z tel que ?
    Je veux bien faire l’effort de comprendre que p et q sont des entiers naturels (encore que plus tard on trouve p' = 2Racine(2)) mais quel est le rapport entre p et q d’une part et x’, y’ et z’ d'autre part ? Puis entre les données précédentes et X, Y et Z ? Puis entre les précédentes et x, y, z et N ?
    Je suis Charlie.
    J'affirme péremptoirement que toute affirmation péremptoire est fausse

  13. #12
    leg

    Re : Si Fermat avait la solution..

    bonjour Mediat
    Fermat,suppose où trouver p et q ?
    formant un triplet de racines carrées x',y' et z' tel que
    x'² +y'² = z'² equivalent X+Y = Z ou xN+yN=zN

    l'idée de fermat c'est de trouver un couple de paramètres
    par exemple il existe p et q donnant 3,4 et 5 et il existe aussi p' et q' réel donnant 1+8=9 tel que :
    (5+4) (5-4)= (p²-q²)²= x²=3² donc p' et q' donne un triplet de racines carrées de 1, de 8 et de 9..etc son idée générale c'est de montrer qu'il n'existe pas un couple de réel p' et q' paramètrant un triplet primitif
    ("un triangle rectangle, tout d'abord pour N1 et 2 différent de ceux donné par p et q entiers; si il en existait un il est évident que la formule des triplets pythagoricien ne donne pas toutes les solutions entières ce qui est absurde!")
    1) dans les entiers à la puissance N =1
    2) dans les racines carrées des produits à la puissance N>1
    3) cela lui faisait obligation de démontrer l'absence de solution quelque soit une puissance N paire >2, dans un premier temps. puis il ne lui réstait plus qu'à montrer qu'il n'existe non plus un couple de praramètres p' et q' formant un triplet de racines carrées algébriques dans les entiers à la puissance N=3 ou N premiers.
    en démontrant qu'il n'existe pas de couple de réels p' et q' formant un triplet primitif de racines carrées dans les puissances N=1 ,2 4, 6 et 3 ce qui est vrai pour N =1,2 et 3 est vrai pour tout N
    afin qu'il n'y ai pas confusion: un triplet dans N=1 donne solution dans N=2
    un triplet dans N=2 donne solution dans N=4 et N=2
    un triplet dans N = 3 donne solution dans N=6,N=2 mais aussi N=3
    4)il pouvait aussi démontrer pour N=3 qu'il n'exsite pas de solution tel que la différence entre deux cubes ne peut être un demi cube ou le double d'un cube ce qui est pareil. car cela impliquerait x3+y3 =z3
    du fait que la solution de (z+y)(z-y) donnerait u3et v3 et où :
    u3 - v3 donne 2y donc si il existerait cette supposition, deux cube et un demi cube ; en multipliant par 2 cet y on retrouverait une solution cubique, du fait qu'il existerait un couple p' et q' paramétrant un triplet de racines carrée de cubes, c'est à dire racine de u au cube,racine de v au cube et bien sur 2 demis cubes = un cube donc racine de y' au cube!
    En gros une infinité de solutions dans N=3 avec une infinité de multiples...
    et ceci se généralise par la méthode des triplets pythagoricien...
    5)Fermat ne pouvait pas démontrer cas par cas l'absence de solution dans une puissance première > à 2 et 3 ce qui est impossible;
    d'où sa méthode de descente infinie ne peut être utile pour démontrer le cas de tous N premiers!

  14. #13
    inviteeb9ddbba

    Re : Si Fermat avait la solution..

    Citation Envoyé par leg Voir le message
    bonjour Mediat
    Fermat,suppose où trouver p et q ?
    formant un triplet de racines carrées x',y' et z' tel que
    x'² +y'² = z'² equivalent X+Y = Z ou xN+yN=zN

    euh non toujours pas compris

  15. #14
    leg

    Re : Si Fermat avait la solution..

    Citation Envoyé par jahlucine Voir le message
    euh non toujours pas compris
    pour toi qu'est-ce que c'est qu'un triplet pythagoricien?
    et qu'elle formule les donnent tous? (restons avec les primitifs)

    pour Mediat:
    en complément: Fermat à du s'apercevoir en démontrant le cas N=4, qu'ils n'existait pas un couple de paramètres p' et q' réels formant un triplet pythagoricien primitif d'entiers naturels car il lui suffisait de trouver deux carrés et un entier et si ce triplet est solution, alors le cas N=4 est faux ainsi que son G théorème.
    son raisonnement devient alors si p' et q' ne peuvent paramétrer un triplet qui est solution de N=4 ou du moins x² + (y²)² = (z²)² il ne peut exister d'avantage un triplet de trois cubes , ou de deux cubes et un demi cube ou encore 2 cubes et la racine carrée d'un cube.
    les démos du cas N=4 se généralisent à toutes les puissances N pair >2 ...

  16. #15
    prgasp77

    Re : Si Fermat avait la solution..

    leg, on ne critique pas l'idée, mais la manière de la présenter. Une démonstration ne peut être décidée comme correcte ou incorrecte si elle n'est pas présentée de manière rigoureuse.

    Quelques conseils :
    > À chaque symbole introduit, précise ce qu'il représente, de quel ensemble il est élément, s'il est connu/inconnu, fixé/variable etc.
    > Précise à chaque fois les théorème utilisés
    > Explique avant chaque équation ce que tu tentes de faire.
    > Numérote tes équations pour y faire allusion plus tard.

    Bonne chance;
    gasp qui n'a pas tout compris ...
    --Yankel Scialom

  17. #16
    martini_bird

    Re : Si Fermat avait la solution..

    Salut,

    juste un message pour rendre hommage à notre leg national !

    En effet, je lis depuis mon inscription ses nombreuses tentatives de démonstration du grand théorème de Fermat à partir des triplets pythagoriciens... Et bon j'abandonne assez vite car je ne comprends pas tout...

    Mais je promets de faire un effort pour l'occasion, étant donné que tu nous a présenté la démo dans un beau pdf.

    Amicalement.

    PS : ça fait combien de temps que tu es dessus ?
    « Angle éternel, la terre et le ciel, pour bissectrice, le vent. » Garcia Lorca

  18. #17
    ericcc

    Re : Si Fermat avait la solution..

    Habituellement après les post de Leg, on voit arriver Pole et g_h qui semblent entamer un dialogue assez fructueux avec lui. Que se passe t il donc ?

  19. #18
    leg

    Re : Si Fermat avait la solution..

    bonjour Martini
    ce que tu dis est vrai, mais je n'y consacre pas mon temps, quelques heures par semaine travail oblige .
    j'ai commencé à m'interresser aux triplets Py en 2000 puis à Fermat et son G.T en 2002 à peu prés, avec une première ébauche en 2004 car j'avais trouvé assez vite les deux démos du cas N = 4 x et z carré avec y et z carré , en lisant la démo de X et Y carré que j'avais résolu, sans la rigueur actuelle qui est demandée; mais comme avait pu le faire Fermat à son époque:
    uniquement avec la formule des Triplets Py.

    car l'anexe A en page 7 et 8 du pdf comporte une contradiction trés élémentaire qui permet de conclure le cas N=4 et dont l'auteur ainsi que de nombreux mathéux sont passé au travers, pour me faire dire comment personne n'a pu publier ces deux démos ou théorèmes pour faire plaisirs a prgasp77
    Cas N = 4:
    si dans un triplet pythagoricien, X est un carré Y ne peut en être un! (1) largement difusé

    si dans un triplet pythagoricien, X est un carré Z ne peut en être un! (2) jamais publié

    si dans un triplet pythagoricien, y est un carré Z ne peut en être un! (3)
    jamais publié

    ce qui n'est pas mon but personnel mais de croire que Fermat à son époque pouvait l'avoir démontrer avec un succession de raisonnement par l'absurde:
    du style:
    si deux réel p' et q' ne peuvent paramétrer un triplet Py primitif de racines carrées qui ne serait donné par p et q entiers alors il est évident que son théorème n'a pas de solution!
    et lorsque j'ai posé la question à plusieur reprise sur ce site ou à des matheux dont, de trés trés bon, la réponse est unanime mais c'est idiot il en existe une infinité.

    j'ai répondu oui, mais des multiples de primitif !

    ce qui n'apporte rient, car on les obtiens toujours pareil avec deux entiers p et q est un multiple commun!
    ("trouve en un seul prgasp77, mais peut être qu'il faut un peu plus d'effort pour comprendre !")

    et pour démontrer ceci il faut commencer par les puissances paires, car le triplet si il existe, il est forcément constitué de 3 entiers : 3 carrés, 3cubes ,3entiers à la puissanc N premier >3
    ce qui revient à paramètrer un triangle rectangle primitif avec trois produits de puissance N premier
    en commençant par N=2 sans perte de généralité pour l'équation de Fermat X+Y = Z ou ce qui est équivalent: x'N+ y'N = z'N ; comme cela à était déjà montré sur le dernier sujet ouvert par Gaétan.
    en attendant vos remarques, et merci à toi Martini pour les prochaines questions que

  20. #19
    leg

    Re : Si Fermat avait la solution..

    Citation Envoyé par ericcc Voir le message
    Habituellement après les post de Leg, on voit arriver Pole et g_h qui semblent entamer un dialogue assez fructueux avec lui. Que se passe t il donc ?
    il font comme toi ericcc il réfléchisse et se demande si c'est vrai

  21. #20
    ericcc

    Re : Si Fermat avait la solution..

    Citation Envoyé par leg Voir le message
    il font comme toi ericcc il réfléchisse et se demande si c'est vrai
    Pour ça je peux te répondre : Fermat avait RAISON ! Andrew Wiles l'a démontré.

  22. #21
    leg

    Re : Si Fermat avait la solution..

    Citation Envoyé par ericcc Voir le message
    Pour ça je peux te répondre : Fermat avait RAISON ! Andrew Wiles l'a démontré.
    Wiles, à démontré ce que Fermat à eu comme idée il n'existe pas un couple de réel p' et q' paramétrant un triplet PY c'est à dire un triangle rectangle primitif mesuré par des racines carrées d'entiers élevé à la puissance N premièr > ou = 2!
    ni pour N =1 qui dans ce cas précis serait différent de ce donné par p et q entiers non nul
    ni par p et q et un facteur k commun, entier non nul >1 tel que
    k p²-q² = x' ; k 2pq = y' et k p²+q² = z' et qui dans ce cas, indique qu'il s'agit d'un triplet multiple d'un primitif

  23. #22
    inviteeb9ddbba

    Re : Si Fermat avait la solution..

    Citation Envoyé par leg Voir le message
    Wiles, à démontré ce que Fermat à eu comme idée il n'existe pas un couple de réel p' et q' paramétrant un triplet PY c'est à dire un triangle rectangle primitif mesuré par des racines carrées d'entiers élevé à la puissance N premièr > ou = 2!
    ni pour N =1 qui dans ce cas précis serait différent de ce donné par p et q entiers non nul
    ni par p et q et un facteur k commun, entier non nul >1 tel que
    k p²-q² = x' ; k 2pq = y' et k p²+q² = z' et qui dans ce cas, indique qu'il s'agit d'un triplet multiple d'un primitif
    Leg je ne remets pas en question ton habilete en arithmetique mais enfin c'est pas possible de rediger aussi mal. le francais est vraiment lamentable et je pense que si tu veux qu on te donne notre avis il faut redefinir les notions .... meme celles qui te semblent triviales a toi ... car souvent c'est la que se cachent les erreurs de raisonnement

  24. #23
    leg

    Re : Si Fermat avait la solution..

    Citation Envoyé par jahlucine Voir le message
    Leg je ne remets pas en question ton habilete en arithmetique mais enfin c'est pas possible de rediger aussi mal. le francais est vraiment lamentable et je pense que si tu veux qu on te donne notre avis il faut redefinir les notions .... meme celles qui te semblent triviales a toi ... car souvent c'est la que se cachent les erreurs de raisonnement
    bonjour jahlucine
    il faudrait que tu me dise exactement ce que tu ne comprend pas,
    a) dans le fond du raisonnemnt, ou le fait que j'utilise la relation de pythagore et sa formule pour montreren premier l'absence de solution dans N = 4)
    1) l'absence de solution dans N pair > 2 de façon générale
    2) dans N=3 puis je généralise.

    je revien donc sur cette relation de pythagore, qui à déjà était montré dans le fil de gaétant, et dont zinia est intervenu pour confirmer que cette méthode n'enlevait aucune généralité sur l'équation de fermat

    3): x² +y² =z² est une équation de Fermat pour N=2 et vérifie le théorème de pythagore! x,y et z sont trois racines carrées c'est a dire trois entiers à la puissance N=1 paramétré par la formule des triplets Pyt

    4) X+Y = Z sont une relation de pythagore et une équation de fermat pour N=1; en aucun cas cette équation ne peut être un triplet Py, c'est à dire un triangle rectangle car la somme des côtés est = à l'ypothénuse cela formerait un angle plat 180°;
    et dont les trois racines carrées de cette équation sont x',y' et z' ; toujours paramétré par la même formule des T. py, il est évident de constater que ces racines peuvent être 3 entiers au maximum, ou 3 réel algébriques donc le couple de paramètres est soit 2 entiers ou 2 réels...

    5) x3+ y3= z3est aussi une relation de pythagore est une équation de Fermat; soit uniquement pour N=3 ou pour les deux, N=3 et 6 (pour n'en rester qu'aux cubes)
    les 3 racines carrées de cette équation sont toujours paramètrées par la même formule des T.Py, et ces 3 racines mises au carré vérifient bien les deux relations : pythagore et Fermat sans perte de généralité !
    le couple de paramètres est forcément par évidence soit deux réels, algébrique ou non dans ce cas la solution est unniquement dans N = 3; au moins une de ces racine n'est un cube au minimum et au maxi les trois racines carrées de cette équation ne sont des entiers donc des cubes; ce ne pourrait être qu'une solution, uniquement dans N=3!
    le couples de paramètres sont deux entiers, les trois racines sont des entiers, donc des cubes (toujours par supposition) la solution se vérifie l'équation de Fermat pour N = 3 et 6 mais aussi N = 2 , le triangle rectangle étant mesuré par trois cubes, il vérifie bien l'équation de pythagore en mettant ces trois cubes au carré!
    le carré de l'ypothénuse = la somme des carrés des deux côtés du triangle rectangle!

    je ne vois vraiment pas, ce qu'il y a de difficile à comprendre,
    et je le répète, P de Fermat ne disposait que de ces outils mathèmatique à son époque alors il faudra faire avec.
    peut être que c'est le fait, que personne n'est réussi à démontrer le cas N=4, uniquement avec ces deux propriétés à savoir: (a) si dans un triplet Py ; X est un carré alors Z ne peut en être un !
    (b) si dans un triplet Py Y est un carré alors Z ne peut en être un! d'où on en conclu l'absence de solution dans N = 4 telque X4+ Y4= Z4
    que cela rende difficile à comprendre ma méthode, qui risque de s'averrait être la méthode de Fermat.


    ("j'éspère pour vous qu'un jour, on ne vous colle pas ces deux exercices, car il y aurra des surprise dans le nombre d'échec..")

    en espèrant que ces quelques explications te permettrons d'y voir plus clair, dans le principe.
    A+

  25. #24
    leg

    Re : Si Fermat avait la solution..

    pour en revenir un peu sur cette histoire, Willes a du expliquer sa démo avec 200 pages et avec des outils mathématiques trés complexes et totalement inexistant en 1640 .
    mais surtout en transformant l'équation de fermat en une autre équation, lui permettant de démontrer un conjecture qui englobe celle de Fermat .

    Son explication dans son intégralité est comprise par combien d'entre vous? (je parle de personne faisant des math à longueur de journée ou dont c'est la proffession ..etc)

    combien de personne sont là pour dire: une équation de pythagore ce n'est pas une équation de Fermat et j'ai répondu à chaque fois: pourquoi ?
    un entier qui est la somme de deux entiers n'est ce pas un triangle rectangle dont j'ai mis les trois côtés au carré, qui vérifie cett équation et le théorème de pythagore tel que:
    (p²-q²)² + (2pq)² = (p²+q²)²

    est il impossible de former ce triangle rectangle avec des produits de puissance ou leur racine carrées ?

    p² + q² = 2pq + (p-q)² n'est ce pas connu depuis des siecles? qui rende si difficile à comprendre que ce corollaire implique l'absence de solution dans N=4
    uniquement en se disant que si z et y sont deux carrés
    en sachant que la différence d entre z et y est toujours un carré (p-q)² = d²
    alors que la différnce entre Z² et Y² = X² = (p²-q²)² où X est le côté opposé à l'hypoténuse Z.
    et vous penseriez que Fermat spécialiste de pythagore ne s'en est pas rendu compte , cela serait le prendre pour un nul ou un néophite qui racontait n'importe quoi.

  26. #25
    inviteeb9ddbba

    Re : Si Fermat avait la solution..

    Citation Envoyé par leg Voir le message

    5) x3+ y3= z3est aussi une relation de pythagore (...)
    les 3 racines carrées de cette équation sont toujours paramètrées par la même formule des T.Py, et ces 3 racines mises au carré vérifient bien les deux relations : pythagore et Fermat sans perte de généralité !

    le couple de paramètres est forcément par évidence soit deux réels, algébrique ou non dans ce cas la solution est unniquement dans N = 3; au moins une de ces racine n'est un cube au minimum et au maxi les trois racines carrées de cette équation ne sont des entiers donc des cubes; ce ne pourrait être qu'une solution, uniquement dans N=3!
    le couples de paramètres sont deux entiers, les trois racines sont des entiers, donc des cubes (toujours par supposition) la solution se vérifie l'équation de Fermat pour N = 3 et 6 mais aussi N = 2 , le triangle rectangle étant mesuré par trois cubes, il vérifie bien l'équation de pythagore en mettant ces trois cubes au carré!
    le carré de l'ypothénuse = la somme des carrés des deux côtés du triangle rectangle!
    pffffff ....

    leg je suis vraiment frustre car je suis convaincu que vous dites des choses interessantes mais vraiment j'essaie mais je comprends rien. Les racines dont vous parlez sont elle cubiques ? carrees ? Les relations de fermat et de pythagore dont vous parlez ca vous tuerait de nous les reecrire? Ces 2 bonhommes ont tellement de relations ...

    Enfin vous etes pas matheux mais si vous avez lu la demo de Wiles vous devez connaitre les conventions simples d'ecriture en maths.

    Ecrivez par exemple :

    si x2+y2=z2 avec (x,y,z) E N2

    Alors on a x= .... donc y = ....

    En utilisant la relation pythagore (...) on obtient

    ...=.... donc ...

    C'est pas sorcier et ca vous aidera vous meme a cerner tout ca.

    La prose en maths c'est rarement efficace...

  27. #26
    leg

    Re : Si Fermat avait la solution..

    jahlucine
    (je n'ai pas lu la démo de Willes j'en serait totalement incapable)
    mais du fait que j'utilise les triplets pythagoriciens avec leur formule il ne peut être question que de racines carrées et non de racines cubiques donc x' est une racine carrée = p²-q²
    pour N= 6 , x^3 est une racine carrée = p² - q²
    pour N = 4, x² est une racine carrée = p² - q²
    pour N=3, x' est une racine carrée non entières ,algébrique = p² - q² si et seulement si X^3n'est pas un carré parfait (ex 8 ou 64 soit 2^3 ou 4^3 ce qui donnerait comme racine carrée x' ou 8)
    tout n'est que racine carrées lorsque je parle de racine!

    je comprend ton effort et t'en remerci mais je n'ai jamais fais ni appris les formules rigoureuses en math !

    une équation de Fermat c'est xN+YN=zN pour N >1
    une relation de pythagore qui vérifie une équation de Fermat c'est :
    (p² - q²)² = xN donc yN = (2pq)²
    et zN = (p²+q²)²

    ce qui à pour résultat que chaque triplet de racines carrées vérifie la relation ou si tu préfère le théorème de pythagore et l'équation de Fermat
    puisque j'utlise uniquement la formule qui donne les triplets pythagoriciens entiers puis non entiers mais c'est toujours la même formule elle ne m'interdit pas de former des triangle rectangle quelque soit les valeurs.

    ("voila pourquoi il n'y a pas de perte de généralité pour le theorème de fermat en utilisant cette relation Pythagorique")

    par ex: je te dis 4 - 1 = 3 quel est la valeur des 3 côtés de ce tiangle rectangle ou de ce triplet Pyt,
    et quel est la valeur de p' et q' qui ne peuvent être des entiers ?
    sachant que p' - q' =d et p'² -q'² = x' = sqrt de 3 et que x' = d² + (dn) et où (dn)/d =n et n/2 = q'
    et bien entendu q' + d = p'

    voila pour t'eviter de chercher,
    P’ = 1.366025403….... , q’ =0.366025403….....

    une équation de fermat pour N=1, c'est X + Y = Z où: sqrt de X = p² - q² pour Y = 2pq et pour Z = p²+q²
    donc avec l'exemple si dessus, si tu as 3 + 1 = 4 cela donne
    (p² -q²)² + (2pq)² = (p² + q²)² qui est égale à : X+Y =Z, d'où le carré de l'hypoténuse = la somme des carrés des côtés de l'angle droit, équivalent à l'équation de fermat 1+3=4 dans N=1
    voila, c'est de l'algèbre élémentaire de mon époque et qui en plus, que je n'ai jamais eu l'occasion d'aprendre..je suis parti avec mon cep à 13 ans et demi en 1961 ..

    imagine alors moi les efforts que je dois faire pour comprendre vos formules mathématiques que je décortique tant bien que mal...

  28. #27
    leg

    Re : Si Fermat avait la solution..

    encore une petite chose à ta question:
    si x2+y2=z2 avec (x,y,z) E N2
    est ce que cela veut dire que (x,y,z) appartiennent aux entiers naturels?

  29. #28
    ericcc

    Re : Si Fermat avait la solution..

    Leg I believe you are pulling ours

  30. #29
    leg

    Re : Si Fermat avait la solution..

    Citation Envoyé par ericcc Voir le message
    Leg I believe you are pulling ours
    qu'est ce que je suis ?

  31. #30
    leg

    Re : Si Fermat avait la solution..

    Bonjour a tous

    Ce petit résumé du pdf, devrait vous permettre de comprendre l’idée générale et ce que P de Fermat à du faire, (cela pourrait presque s’écrire dans la marge de son Diophante..)

    T1 :
    Si X est un carré, alors Z ne peut en être un !

    De T1 , on en tirera T2 : Si Y est un carré, alors Z ne peut en être un !)

    Preuve :
    (« Triplet primitif, où le facteur commun K = 1 »)

    Si p² - q² = x² , 2pq = y² , et p² + q² = z²

    A)
    Nous obtenons : p² - q² = x² où p, q est x sont pythagoriques
    Ainsi que : p² + q² = z² où p , q est z sont aussi pythagoriques
    Il existe donc :
    p1 et q1, tel que : 2 p1 q1 = q ;
    ainsi que p2 et q2 , tel que : 2 p2 q2 = q
    d’où p1 = p2 = p 3 et q1 = q2 = q3
    c'est-à-dire un couple p3 et q3 qui aurait paramètré ces deux triplets, ce qui est absurde.

    B)
    Ceci nous fait remarquer que dans cette supposition on aurait aussi z = x alors que p = x ou z mais pas les deux du fait que p , q et x ou z ; sont pythagoriques ! (ce qui est aussi impossible)

    C)
    De part la propriété des triplets pythagoriciens, on a aussi :
    (p² - q²)² = ((p² + q²) + (2pq)) ((p² + q²) - (2pq))
    Donc on obtient :
    (x²)² = (z² + y²) (z² - y² ) ? (du fait que le triplet est par supposition, constitué de 3 carrés)
    Ce qui est contradictoire, on se retrouve dans le même cas que A) !
    On pourrait dans ce cas remplace z et y par : p4 et q4 (« avec une descente infinie d’entiers, c’est à dires de racines carrées ! ») On en conclu que T2 est vrai !

    Quel raisonnement R on en tire ?
    R1)
    Tout simplement, « l’idée de Fermat.. » En effet, si cela existe, alors par obligation il existe deux couples de réels, p’ et q’ ainsi que p’’ et q’’, qui ont paramètré les deux triplets de A),
    A savoir (p,q , x ) avec (p,q, z) ; mais il existe aussi deux autres couples de réels p’’’ et q’’’ ainsi que p’’’’ et q’’’’ ; qui ont paramétré les deux solutions : (z² + y²) = u² et (z² - y² ) = v² du fait que (u² v²) = (x²)² par supposition et contradiction C) et A) !

    R2)
    Si c’est 4 couples de réels existent, alors le couple d’entiers p et q , donnant le triplet primitif existe aussi ! ce qui est complètement absurde !
    (z² + y²) (z² - y² ) ≠ (x²)² , le produit de deux carrés parfaits premiers entre eux deux à deux ne peut être un carré !
    Ou alors le cas N = 4 est faux, ce qui implique que notre ami A willes , a vraiment perdu du temps pour montrer qu’il ne peut exister un couple de réel pouvant paramétrer un triplet primitif,
    On en conclu qu’un couple de réel p’ et q’ ne peut paramétrer un triplet Pyth primitif, qui ne serait donné par p et q entiers, tel que défini dans le pdf,
    Par conséquent, p’ et q’ réels, ne donne un triplet primitif dans les puissance N =1,2 et 4 pas de solution pour N = 4
    (« le contraire implique que la formule des triplets pythagoricien ne donne pas toutes les solutions entières, ce que l’on saurait depuis des siècles »)

    T3
    Si X est un cube, alors Z ne peut être un cube, de T3 on en retirera T4 : si Z est un cube alors Y ne peut en être un
    Si p² - q² = x3 , 2pq = y3 et p² + q² = z3 ceci nous ramène à la contradiction C) du cas N = 4

    (« je passe sur le fait que je pourrai choisir p et q dans les cubes ou leur racines carrées et qui mis au carré me donnerait la contradiction A) et B) tel que (p3)² - (p3)² = x3 avec (p3)² + (p3)² = z3 ainsi que 2 p3 q3 = y3 dans ce cas précis, si p et q sont deux cubes , y au cube ne peut exister ce qui serait contraire à la supposition ceci implique que p serait un cube, et q un demi cube pair ce qui donnerait bien 2 p3 (q3 /2) = y3 »)
    Donc :
    C1)
    (p² - q²)² = ((p² + q²) + (2pq)) ((p² + q²) - (2pq)), soit :

    (x3)² = (z3 + y3) (z3 – y3 ) on va retrouver la contradiction du cas N = 4 !

    R2)
    Il existe alors deux couples de réels p’ et q’ avec p’’ et q’’ qui ont paramétré ces deux solutions (z3 + y3) = (u3)² ; (z3 – y3 ) = (v3)² car : (x3)² = ((u3)² (v3)²)
    Donc il existe bien un couple d’entiers p et q choisis ou non dans les racines carrées des cubes ayant donné ce triplet primitif de cube ce qui est absurde,
    Et il existerait aussi deux triplets primitifs dans les racines carrées entières ou non correspondant à ces deux solutions, tel que √ z3 ,√ y3 , √(u3)² avec √ z3 ,√ y3 , √(v3
    Ce qui est impossible et contradictoire !
    R3
    (« Euler s’est planté, sa démonstration est fausse, ainsi que A Willes pour la deuxième fois (L Euler ne savait pas qu’il existait des réels) »)
    Par conséquent p’ et q’ réels, ne peuvent donner un triplet primitif dans une puissance N = 1,2,3,4 et 6 qui ne serait donné par p et q entiers, ou racines carrées algébriques de produits cubiques tel que: sqrt de 8 et 27 par exemple c’est à dire deux racines carrées de cubes, non carrés !
    Je pense qu’il est inutile de pousser jusqu’à S. Germain pour N = 5 ce qui est vrai pour N = 1,2 ,3.. l’est pour tout N.

Page 1 sur 3 12 DernièreDernière

Discussions similaires

  1. Dingue! Einstein avait encore raison!
    Par invitecb70ab37 dans le forum Physique
    Réponses: 43
    Dernier message: 05/07/2004, 18h39